Đến nội dung

phudinhgioihan nội dung

Có 329 mục bởi phudinhgioihan (Tìm giới hạn từ 24-05-2020)



Sắp theo                Sắp xếp  

#379030 Tìm giới hạn bằng vô cùng bé tương đương

Đã gửi bởi phudinhgioihan on 20-12-2012 - 11:34 trong Giải tích

Các bạn giải giúp mình bài này bằng VCB với
$ \lim_{x\to 0}\frac{x-sin5x+sin^2x}{4x+arcsin^2x+x^2} $


Thật ra cũng chẳng cần VCB làm gì, nhưng nếu VCB thì:

$\lim_{x\to 0}\frac{x-sin5x+sin^2x}{4x+arcsin^2x+x^2}=\lim_{x \to 0} \dfrac{1-\frac{\sin 5x}{x}+\frac{\sin^2x}{x}}{4+\frac{\arcsin^2x}{x}+x}$

Ta biết rằng

$\frac{\sin x}{x} \underset{x \to 0}\sim 1 $

$\frac{\arcsin x}{x} \underset{ x \to 0} \sim 1$

$\Rightarrow \dfrac{1-\frac{\sin 5x}{x}+\frac{\sin^2x}{x}}{4+\frac{\arcsin^2x}{x}+x} \underset{x \to 0} \sim \dfrac{1-5+\sin x}{4+\arcsin x}\underset{x \to 0}\sim -1 $

$\Rightarrow \lim_{x\to 0}\frac{x-sin5x+sin^2x}{4x+arcsin^2x+x^2}=-1 $



#382215 Ảnh thành viên

Đã gửi bởi phudinhgioihan on 31-12-2012 - 11:15 trong Góc giao lưu

Có chú nào muốn làm em rể giơ tay lên, giơ chân luôn nhé :)) Hình đã gửi



#398059 Topic về Bất đẳng thức trong Tích phân

Đã gửi bởi phudinhgioihan on 18-02-2013 - 20:10 trong Giải tích

Bài 45:
b) Hàm $f(x)$ khả tích trên toạn $[0;1]$ và $\int_0^1 f(x)dx >0$. Chứng minh tồn tại đoạn $[a;b]\subset [0;1]$ mà trên đó $f(x)>0$


Giả sử $\int_0^1 f(x)dx=I>0$

Với $n \in \mathbb{N}^*$, xét phân hoạch $P$ chia $[0;1]$ bởi các điểm $\frac{i}{n}\;, i =\overline{0,n}$, ta có

$$I=\lim_{n \to +\infty} \frac{1}{n}\sum_{i=0}^n f(\frac{i}{n})$$

$$\Leftrightarrow \forall \epsilon>0, \exists n_0>0|\; n>n_0 \Rightarrow \left| \frac{1}{n}\sum_{i=0}^n f(\frac{i}{n}) -I \right| <\epsilon$$

Chọn một $0<\epsilon_0<l$ để cố định, khi đó

$$\exists n_0>0|\; n>n_0 ,\; \left| \frac{1}{n}\sum_{i=0}^n f(\frac{i}{n}) -I \right| <\epsilon_0$$

$$\Rightarrow \exists n_0>0|\; n>n_0 , \frac{1}{n}\sum_{i=0}^n f(\frac{i}{n})>I-\epsilon_0>0$$

Do đó phải tồn tại $i_0 \in \mathbb{N}$ sao cho $f(\frac{i_0}{n})>0$. Do $f$ liên tục trên $[0;1]$ nên tồn tại một $\lambda-$lân cận của $\frac{i_0}{n}$ sao cho $f(x)>0 \;, \forall x \in (\frac{i_0}{n}-\lambda;\frac{i_0}{n}+\lambda)$

Do $\dfrac{i_0}{n} \in [0;1]$ nên $(\frac{i_0}{n}-\lambda;\frac{i_0}{n}+\lambda) \cap [0;1]$ là một đoạn, suy ra đpcm.



#394062 Topic về Bất đẳng thức trong Tích phân

Đã gửi bởi phudinhgioihan on 06-02-2013 - 21:32 trong Giải tích

Bài 37: Cho $[a;b] \subset \mathbb{R} \;, f:[a;b] \to \mathbb{R}$ khả vi liên tục trên $[a;b]$ sao cho $f(a)=f(b)=0$. $\forall x \in [a;b]$ đặt $g(x)=\min \{|x-a|,|x-b| \} \;, $. Chứng minh

$$\int_a^b \dfrac{f^2(x)}{g^2(x)}dx\le 4\int_a^b f'^2(x)dx$$



#393196 Topic về Bất đẳng thức trong Tích phân

Đã gửi bởi phudinhgioihan on 04-02-2013 - 20:48 trong Giải tích

Bài 30: Cho $f$ không âm, liên tục và đồng biến trên $[0;c] (c>0)$. Chứng minh rằng $$\int_0^a f(x)dx+\int_{f(0)}^bf^{-1}(y)dy \ge ab$$ Với $a\in [0;c];b\in [f(0);c]$

Bài 31: Chứng minh $$e^x-1 <\int_0^x \sqrt{e^{2t}+e^{-t}}<\sqrt{(e^x-1)(e^x-\frac{1}{2})}\;\; \forall x>0$$

Bài 32: Tìm giá trị nhỏ nhất của $$f(n)=\int_{\frac{\pi}{4}}^{\frac{3\pi}{8}}\left(\frac{\sin^{n+2}x}{\cos^n x} +\frac{\cos^{n+2}}{\sin^nx}\right )dx,n\in \mathbb{Z}^+$$

Bài 33: Cho $m\in \mathbb{N}$. Tìm giá trị nhỏ nhất của biểu thức $$f(x)=\int_1^x t^m.e^{2t}dt-2\left(\frac{x^{m+3}}{m+3}+\frac{x^{m+2}}{m+2} \right ),\, x\ge 1$$


Bài 30: Đã chứng minh ở đây. Đây là bất đẳng thức Young

Bài 31:

Hiển nhiên $$\int_0^x \sqrt{e^{2t}+e^{-t}}> \int_0^x e^t=e^x-1$$

Đặt $$f(x)=\int_0^x \sqrt{e^{2t}+e^{-t}}-\sqrt{(e^x-1)(e^x-\frac{1}{2})} \;\;, x>0$$

$$f'(x)=\sqrt{e^2x+e^{-x}}-\dfrac{e^x}{2} \left(\sqrt{\dfrac{e^2-1}{e^x-\frac{1}{2}}} +\sqrt{\dfrac{e^x-\frac{1}{2}}{e^x-1}}\right)$$

Đặt $t=e^x >1 $

Xét bất phương trình $f'(x)<0$

$$\Leftrightarrow \sqrt{t^2+\frac{1}{t}}<\dfrac{t}{2}\left( \sqrt{t-\dfrac{1}{2(t-\frac{1}{2})}}+\sqrt{1+\dfrac{1}{2(t-1)}} \right)$$

$$\Leftrightarrow \dfrac{8}{t^3}<\dfrac{1}{t-1}+\dfrac{1}{t-\frac{1}{2}}$$

$$\Leftrightarrow 4t^4+24t-3t^3-16t^2-8>0$$

Luôn đúng $\forall t>1$

Vậy $f'(x)<0 \;\;, \forall x>0$ , suy ra $f(x)$ nghịch biến trên $(0;+\infty)$

do đó $f(x)<f(0)=0$ , vậy có đpcm.

Bài 32:

Do $x \in [\dfrac{\pi}{4}, \dfrac{3\pi}{8}] $ thì $\sin x >0 \;, \cos x >0$ nên ta có thể thác triển $f$ lên $\mathbb{R}$

Xét $f^1(n)=\int_{\frac{\pi}{4}}^{\frac{3\pi}{8}}\left(\frac{\sin^{n+2}x}{\cos^n x} +\frac{\cos^{n+2}}{\sin^nx}\right )dx \;, n \in \mathbb{R}_+$

$${f^{1}}^{'}(n)=\int_{\frac{\pi}{4}}^{\frac{3\pi}{8}} \ln \tan x \left( \frac{\sin^{n+2}x}{\cos^n x} -\frac{\cos^{n+2}}{\sin^nx}\right )dx $$

$$\forall x \in [\dfrac{\pi}{4}, \dfrac{3\pi}{8}] ,\; \ln \tan x \ge 0 \;, \sin x \ge \cos x$$

$$\Rightarrow \frac{\sin^{n+2}x}{\cos^n x} -\frac{\cos^{n+2}}{\sin^nx} \ge \cos^2 x-\cos^2 x=0$$

$$\Rightarrow {f^{1}}^{'}(n) \ge 0$$

Vậy $f^1(n)$ đồng biến trên $[0+\infty)$ , suy ra $f^1(n) \ge f^1(0)=\dfrac{\pi}{8}$

Suy ra $f(n) \ge f(0) \dfrac{\pi}{8}$

Bài 33:

$$f'(x)=x^me^{2x}-2\left(x^{m+2}+x^{m+1} \right )=x^m\left( e^{2x}-2x-2x^2 \right) $$

Lại có $e^x \ge 1+x+\dfrac{x^2}{2} \;, \forall x \ge 0$

suy ra $e^{2x} >1+2x+2x^2 \;, \forall x \ge 1$

Do đó $f'(x) >x^m>0$ , suy ra $f(x)$ đồng biến trên $[1;+\infty)$

Suy ra $f(x) \ge f(1)=-2\left( \dfrac{1}{m+2}+\dfrac{1}{m+3} \right) $



#398086 Topic về Bất đẳng thức trong Tích phân

Đã gửi bởi phudinhgioihan on 18-02-2013 - 20:50 trong Giải tích

Mình nghĩ để ai đó đưa ra lời giải cho hai bài toán trên, vì bài mình đưa ra có dạng tương tự :lol:

Bài 49. [Đặng Thành Nam] Cho $f(x) \ge 0$ là hàm giảm và $f(x) + xf'(x) \ge 0$ với mọi $x \in \left[ {a,b} \right]$. Chứng minh rằng

$$\int\limits_a^b {x{f^2}(x)dx} \le \frac{{b + a}}{{2\left( {b - a} \right)}}{\left( {\int\limits_a^b {f(x)dx} } \right)^2}$$


Với hai hàm thực $f;g$ cùng liên tục trên $[a;b]$, nếu thêm giả thiết một hàm tăng và hàm còn lại giảm thế thì $\forall x;y \in [a;b]$, $(f(x)-f(y))(g(x)-g(y)) \le 0$

$$\Rightarrow \int_0^1\int_0^1 (f(x)-f(y))(g(x)-g(y))dxdy \le 0$$

$$\Leftrightarrow \int_0^1\int_0^1 \left( f(x)g(x)+f(y)g(y)-f(x)g(y)-f(y)g(x) \right) dxdy \le 0$$
$$\Leftrightarrow 2 \left((b-a) \int_0^1 f(x)g(x)dx-\int_a^b f(x)dx\int_a^b g(x)dx \right) \le 0$$

$$\Leftrightarrow \int_0^1 f(x)g(x)dx \le \dfrac{1}{b-a} \int_a^b f(x)dx\int_a^b g(x)dx$$

Quen nhỉ :D

Trở lại vấn đề, ta thấy $f(x) + xf'(x)=(xf(x))' \ge 0$ nên $xf(x)$ là hàm tăng, suy ra

$$\int_a^b xf^2(x)dx \le \dfrac{1}{b-a} \int_a^bxf(x)dx \int_a^b f(x)dx $$

$$\le \dfrac{1}{(b-a)^2} \int_a^bxdx \left(\int_a^b f(x)dx\right)^2 $$

$$\le \dfrac{b+a}{2(b-a)} \left(\int_a^b f(x)dx\right)^2$$



#393202 Topic về Bất đẳng thức trong Tích phân

Đã gửi bởi phudinhgioihan on 04-02-2013 - 21:02 trong Giải tích

Bài 34: (mới chế :D)

Cho $[a;b] \subset \mathbb{R} $ , $f:[a;b] \to \mathbb{R} $ có đạo hàm cấp 2 liên tục trên $[a;b]$ sao cho $f(a)+f(b)=0$. Đặt $m=\min_{x \in [a;b]} f''(x) $, chứng minh

$$\int_a^b f(x)dx \le \dfrac{m(a-b)^3}{12}$$



#398123 Topic về Bất đẳng thức trong Tích phân

Đã gửi bởi phudinhgioihan on 18-02-2013 - 21:40 trong Giải tích

Bài 51:Cho hàm liên tục $f:[0,+\infty)\to [0,1]$ thoả mãn $f(x+y)\leq f(x)f(y),\forall x,y\geq 0$.Chứng minh:$$\int_0^x f(t)dt \geq x\sqrt{f(2x)}.\forall x\geq 0$$


Bài này lừa tình khiếp @@

Do $f(x) \in [0;1] \;, \forall x \ge 0$ nên $f(x+y) \le f(x)f(y) \le f(x) \;\;, \forall x,y \ge 0$

Suy ra $f$ nghịch biến trên $[0;+\infty)$ , suy ra $\int_0^x f(t)dt \ge \int_0^x f(x)dt=xf(x)=x \sqrt{f(x)f(x)} \ge x\sqrt{f(2x)} $



#455355 Topic về Bất đẳng thức trong Tích phân

Đã gửi bởi phudinhgioihan on 05-10-2013 - 19:21 trong Giải tích

Bài 55: Cho $D$ là tập hợp các hàm $f$ biến thực khả vi liên tục trên $[0;1]$ và $f(0)=0\;, f(1)=1 $

 

Đặt $I(f)=\int_0^1 (1+x^2)(f'(x))^2dx $

 

Tính  $\min_{f \in D} I(f) $

 

 




#451701 Topic về Bất đẳng thức trong Tích phân

Đã gửi bởi phudinhgioihan on 19-09-2013 - 19:05 trong Giải tích

Bài 54:

 

Cho $f:[0;1] \to \mathbb{R} $ đồng biến, khả tích trên $[0;1]$ và $f(0) \ge 0$. Chứng minh

 

$$\left(\int_0^1 x^4f(x)dx \right)^2 \le \int_0^1 x^3f(x) dx \int_0^1 x^5f(x)dx \le \dfrac{25}{24} \left( \int_0^1 x^4f(x)dx \right)^2 $$




#450789 Topic về Bất đẳng thức trong Tích phân

Đã gửi bởi phudinhgioihan on 15-09-2013 - 19:37 trong Giải tích

Bài 53: (mới chế :D)

 

a) Cho hai số thực $a<b$ và hàm số $f,g:[a;b]  \to \mathbb{R}$ khả tích trên $[a;b]$ sao cho $\int_a^b f(x)dx \int_a^b g(x)dx=b-a$

 

Chứng minh: $\int_a^b \left(f^2(x)+g^2(x)\right)dx \ge \int_a^b \left(f(x)+g(x) \right)dx $

 

b)  Cho hàm số $f:[0;1] \to \mathbb{R}$ khả tích trên $[0;1]$, chứng minh

 

$$\int_0^1 f^2(x)dx-\int_0^1f(x)dx \ge \dfrac{1}{2} \left[ \left( \int_0^1 f(x)dx \right)^2-1 \right]$$




#450788 Topic về Bất đẳng thức trong Tích phân

Đã gửi bởi phudinhgioihan on 15-09-2013 - 19:28 trong Giải tích


Bài 52: Cho $f(x),g(x)$ là các hàm liên tục trên $[a;b]$ và $0<a\le f(x)\le A; 0<b\le g(x)\le B \forall x\in [a;b]$. Chứng minh BĐT G.Polya

$$\frac{(ab+AB)^2}{4abAB}\ge \frac{\int_a^bg^2(x)dx\int_a^bf^2(x)dx}{\left(\int_a^bf(x)g(x) \right )^2}\ge \frac{4abAB}{(ab+AB)^2}$$

Spoiler

 

 

 

Làm nốt để bắt đầu hâm nóng topic :D

 

Từ giả thiết, dễ có $\dfrac{a}{B}\le \dfrac{f(x)}{g(x)} \le \dfrac{A}{b} $

 

$$\Rightarrow \left( f(x)-\dfrac{a}{B} g(x)\right) \left( f(x)-\dfrac{A}{b} g(x)\right)  \le 0$$

 

$$\Leftrightarrow f^2(x)+\dfrac{aA}{bB}g^2(x)-(\dfrac{a}{B}+\dfrac{A}{b})f(x)g(x) \le 0$$

 

$$\Rightarrow \int_a^b f^2(x)dx+\dfrac{aA}{bB}\int_a^b g^2(x)dx \le (\dfrac{a}{B}+\dfrac{A}{b})\int_a^b f(x)g(x)dx $$

 

Lại có:

 

$$\int_a^b f^2(x)dx+\dfrac{aA}{bB}\int_a^b g^2(x)dx \ge 2\sqrt{\dfrac{aA}{bB}\int_a^b f^2(x)dx \int_a^b g^2(x)dx }$$

 

nên suy ra

 

$$2\sqrt{\dfrac{aA}{bB}\int_a^b f^2(x)dx \int_a^b g^2(x)dx } \le (\dfrac{a}{B}+\dfrac{A}{b})\int_a^b f(x)g(x)dx$$

 

$$\Leftrightarrow 4\dfrac{aA}{bB}\int_a^b f^2(x)dx \int_a^b g^2(x)dx \le \dfrac{(ab+AB)^2}{b^2B^2} \left( \int_a^b f(x)g(x)dx \right)^2$$

 

$$\Leftrightarrow \dfrac{(ab+AB)^2}{4abAB} \ge \dfrac{\int_a^bg^2(x)dx\int_a^bf^2(x)dx}{\left(\int_a^bf(x)g(x) \right )^2}$$

 

Bất đẳng thức

 

 

$$\dfrac{\int_a^bg^2(x)dx\int_a^bf^2(x)dx}{\left(\int_a^bf(x)g(x) \right )^2}\ge \frac{4abAB}{(ab+AB)^2}$$

 

là dễ dàng vì $$\dfrac{\int_a^bg^2(x)dx\int_a^bf^2(x)dx}{\left(\int_a^bf(x)g(x) \right )^2} \ge 1 \ge \dfrac{4abAB}{(ab+AB)^2}$$




#392506 Topic về Bất đẳng thức trong Tích phân

Đã gửi bởi phudinhgioihan on 02-02-2013 - 17:47 trong Giải tích

Bài 29: (Sendov-Skordev)

Cho $f$ không âm trên $[0;1]$ và thỏa $\dfrac{f(x_1)+f(x_2)}{2} \le f(\frac{x_1+x_2}{2}) \;\;, \forall x_1,x_2 \in [0;1]$

Với mọi $n \in \mathbb{N}$, chứng minh

$$\int_0^1 f(x)dx \le 3^n \int_0^1 x^n f(x)dx $$

$$\dfrac{2}{(n+1)(n+2)}\int_0^1 f(x)dx \le \int_0^1 x^n f(x)dx \le \dfrac{2}{n+2} \int_0^1 f(x)dx $$



#390712 Topic về Bất đẳng thức trong Tích phân

Đã gửi bởi phudinhgioihan on 27-01-2013 - 13:15 trong Giải tích

Bài 26:
Cho $\text{f(x)}$ là hàm số liên tục cùng đạo hàm của nó trên đoạn $\text{[a,b]}$ và $\text{f(a) = 0}$. Đặt $\text{M} = \max_{\text{a} \leqslant \text{x} \leqslant \text{b}}\left | \text{f(x)} \right |$.
Chứng minh rằng :
$\text{M}^{2} \leqslant (\text{b} - \text{a})\int_{\text{a}}^{\text{b}}\text{f'}^{2}(\text{x})\text{dx}$.


Lại là Cauchy-Schwarz

$$\forall x \in [a;b] \;, f^2(x) = \left( \int_a^x f'(t)dt \right)^2 $$

$$ \le \int_a^x dt \int_a^x f'^2(t)dt \le \int_a^b dt \int_a^b f'^2(t)dt$$

$$ \le (b-a) \int_a^b f'^2(t)dt$$

Do đó $\max_{x \in [a;b] } f^2(x) \le (b-a) \int_a^b f'^2(t)dt $



#384742 Topic về Bất đẳng thức trong Tích phân

Đã gửi bởi phudinhgioihan on 08-01-2013 - 18:12 trong Giải tích

Bài 21: (Troesch)

Cho hàm thực $h$ lõm, dương có đạo hàm liên tục trên $[0;1] \;\;, h'(0) \ge 0$ . Hàm thực $f$ có $f(0)=0$ và $f'$ liên tục trên $[0;1]$ , chứng minh:

$$\dfrac{\int_0^1h(x)[f'(x)]^2dx}{\int_0^1h(x)dx \int_0^1 [f(x)]^2dx} \ge \dfrac{\pi^2}{4}$$


P/s: Dấu $=$ xảy ra khi và chỉ khi $h$ là hàm hằng, $f(x)=a \sin(\dfrac{\pi x}{2})$

Một trường hợp riêng, chọn $h$ là hàm hằng dương, khi đó ta có:

$$\int_0^1 [f'(x)]^2 dx \ge \dfrac{\pi^2}{4} \int_0^1 [f(x)]^2dx$$



#384571 Topic về Bất đẳng thức trong Tích phân

Đã gửi bởi phudinhgioihan on 07-01-2013 - 23:32 trong Giải tích

Bài 20: (Mới chế :D)

Cho $f : [0;2] \longrightarrow \mathbb{R} $ , $f'$ liên tục trên $[0;2]$ đồng thời $f(2)=0 \;\;, \int_0^2 f(x)dx=\int_0^2 xf(x)dx =k $

Chứng minh : $$\int_0^2 [f'(x)]^2 dx \ge \dfrac{15}{16}k^2$$



#384557 Topic về Bất đẳng thức trong Tích phân

Đã gửi bởi phudinhgioihan on 07-01-2013 - 22:50 trong Giải tích

Phải đánh số thứ tự cho dễ quản lý các bạn ơi!
...................................
Bài 18: Cho f là một hàm số thực nhận giá trị dương và tuần hoàn với chu kỳ bằng 1 trên $\mathbb{R}$. Chứng minh rằng: Với mọi $n=1,2,3,...$ ta luôn có

$\int_{0}^{1}\frac{f(x)}{f(x+\frac{1}{n})}dx\geqslant 1$



$$\int_{0}^{1}\frac{f(x)}{f(x+\frac{1}{n})}dx=\int_0^{\frac{1}{n}}\frac{f(x)}{f(x+\frac{1}{n})} dx+...+\int_{\frac{n-1}{n}}^1 \frac{f(x)}{f(x+\frac{1}{n})}dx $$

$$=\sum_{i=0}^{n-1} \int_{\frac{i}{n}}^{\frac{i+1}{n}} \frac{f(x)}{f(x+\frac{1}{n})}dx$$

Xét $\int_{\frac{i}{n}}^{\frac{i+1}{n}} \frac{f(x)}{f(x+\frac{1}{n})}dx $

Đổi biến $x=t+\dfrac{i}{n}$

$$\Rightarrow \int_{\frac{i}{n}}^{\frac{i+1}{n}} \frac{f(x)}{f(x+\frac{1}{n})}dx=\int_0^{\frac{1}{n}} \dfrac{f(x+\frac{i}{n})}{f(x+\frac{i+1}{n})} dx $$

Do đó : $$\sum_{i=0}^{n-1} \int_{\frac{i}{n}}^{\frac{i+1}{n}} \frac{f(x)}{f(x+\frac{1}{n})}dx=\sum_{i=0}^{n-1}\int_0^{\frac{1}{n}} \dfrac{f(x+\frac{i}{n})}{f(x+\frac{i+1}{n})} dx$$

$$=\int_0^{\frac{1}{n}} (\sum_{i=0}^{n-1} \dfrac{f(x+\frac{i}{n})}{f(x+\frac{i+1}{n})})dx $$

$$ \ge \int_0^{\frac{1}{n}}n \sqrt[n]{\dfrac{f(x)}{f(x+n)}}dx =\int_0^{\frac{1}{n}} n \sqrt{\dfrac{f(x)}{f(x)}}dx=1 $$



#384503 Topic về Bất đẳng thức trong Tích phân

Đã gửi bởi phudinhgioihan on 07-01-2013 - 21:24 trong Giải tích

Cho hàm $f$ thỏa mãn $\left\{ \begin{array}{l}
f \in C\left[ {0;1} \right]\\
xf\left( y \right) + yf\left( x \right) \le 1\,\,\forall x;y \in \left( {0;1} \right)
\end{array} \right.$ Chứng minh:

\[\int\limits_0^1 {f\left( x \right)} dx \le \frac{\pi }{4}\]


Cái giả thiết $\forall x,y \in (0;1) $ và cái chặn trên đặc biệt nên buộc phải nghĩ đến tích phân dính tới lượng giác .

Chọn $y=\sqrt{1-x^2} $
$$\Rightarrow \frac{x}{\sqrt{1-x^2}}f(\sqrt{1-x^2})+f(x) \le \frac{1}{\sqrt{1-x^2}} $$

$$\Rightarrow \int_{0}^1 \frac{x}{\sqrt{1-x^2}}f(\sqrt{1-x^2}) .dx+\int_{0}^1 f(x).dx \le \int_{0}^1 \frac{1}{\sqrt{1-x^2}}.dx $$
$$\Rightarrow - \int_{0}^1 f( \sqrt{1-x^2}).d(\sqrt{1-x^2}) +\int_{0}^1 f(x).dx \le \frac{\pi}{2} $$
$$\Leftrightarrow \int_{0}^1 f(x).dx+\int_{0}^1 f(x).dx \le \frac{\pi}{2} $$



#385004 Topic về Bất đẳng thức trong Tích phân

Đã gửi bởi phudinhgioihan on 09-01-2013 - 16:25 trong Giải tích

Bài 22: Quá đẹp ^^

Cho $n \in \mathbb{N}^*$ ,chứng minh:

$$\sqrt{n} \dfrac{(2n)!!}{(2n+1)!!}\le \int_0^{+\infty}e^{-x^2}dx \le \dfrac{\pi}{2}\sqrt{n} \dfrac{(2n-3)!!}{(2n-2)!!}$$

Hơi khó nên gợi ý : $\int_0^1 (1-x^2)^n = \dfrac{(2n)!!}{(2n+1)!!}$



#385245 Topic về Bất đẳng thức trong Tích phân

Đã gửi bởi phudinhgioihan on 10-01-2013 - 11:20 trong Giải tích

Không biết đúng không sai anh chỉ em với. :D

Ta có:


Theo BDT $Cauchy-Schwarz$


\[\begin{array}{l}{\left( {\int\limits_0^2 {xf\left( x \right)f'\left( x \right)dx} } \right)^2} \le {\left( {\int\limits_0^2 {xf\left( x \right)dx} } \right)^2}{\left( {\int\limits_0^2 {f'\left( x \right)dx} } \right)^2} = {k^2}{\left( {\int\limits_0^2 {f'\left( x \right)dx} } \right)^2}\\
\le 2{k^2}\int\limits_0^2 {{{\left[ {f'\left( x \right)} \right]}^2}dx}
\end{array}\]





Dòng này : $${\left( {\int\limits_0^2 {xf\left( x \right)f'\left( x \right)dx} } \right)^2} \le {\left( {\int\limits_0^2 {xf\left( x \right)dx} } \right)^2}{\left( {\int\limits_0^2 {f'\left( x \right)dx} } \right)^2}$$

Sai rồi em! Đâu có phải C-S đâu. Ráng giải đi, mai thi xong anh post lời giải :D



#390710 Topic về Bất đẳng thức trong Tích phân

Đã gửi bởi phudinhgioihan on 27-01-2013 - 13:10 trong Giải tích

Bài 25 :Cho hàm số $f(x)$ có đạo hàm trên đoạn $[0;1]$ thỏa mãn:

i) $f(0)=f(1)=0$

ii) $\int_0^1 |f'(x)|dx=1$
Chứng minh rằng $|f(x)|\le \frac{1}{2}$ với mọi $x$ thuộc đoạn $[0;1]$


Làm xong rồi cất gối về quê :D


$$\forall x \in [0;\dfrac{1}{2}] \;, |f(x)|= \left| \int_0^x f'(t)dt \right| $$

$$ \le \int_0^x |f'(t)|dt \le \int_0^{\frac{1}{2}} |f'(t)|dt$$


$$\forall x \in [\dfrac{1}{2};1] \;, |f(x)|= \left| \int_{x}^1 f'(t)dt \right| $$

$$ \le \int_x^1 |f'(t)| dt \le \int_{\frac{1}{2}}^1 |f'(t)|dt$$

Suy ra $$ |f(x)| \le \dfrac{1}{2} \left(\int_0^{\frac{1}{2}} |f'(t)|dt+ \int_{\frac{1}{2}}^1 |f'(t)|dt \right)=\frac{1}{2}\int_0^1 |f'(t)dt=\frac{1}{2}$$



#390580 Topic về Bất đẳng thức trong Tích phân

Đã gửi bởi phudinhgioihan on 27-01-2013 - 02:34 trong Giải tích

Bài 24: Cho $f$ là hàm liên tục, có đạo hàm $f'$ liên tục trên đoạn $[0;1]$ và $f(0)=0$. Chứng minh rằng $$\int_0^1|f'(x)f(x)|dx\le \frac{1}{2}\int_0^1|f'(x)|^2dx$$


Buổi sáng tại ShiSha café :D

Ta có:

$$\int_0^1 |f'(x)f(x)|dx =\int_0^1 |f'(x)| \left| \int_0^x f'(t)dt \right| dx $$

$$ \le \int_0^1 |f'(x)| \int_0^x |f'(t)|dt dx $$

$$\le \int_0^1 |f'(x)| \sqrt{\int_0^x dt \int_0^x f'^2(t)dt } dx $$

$$ \le \int_0^1 \sqrt{x} |f'(x)| \sqrt{\int_0^x f'^2(t)dt}dx $$

$$\le \sqrt{\int_0^1 xdx \int_0^1 f'^2(x) \int_0^x f'^2(t)dt dx}$$

$$\le \sqrt{\frac{1}{2} \int_0^1 \frac{1}{2} \left[\left(\int_0^x f'^2(t)dt \right)^2 \right]' dx}$$

$$ \le \frac{1}{2} \int_0^1 f'^2(x)dx$$



#390579 Topic về Bất đẳng thức trong Tích phân

Đã gửi bởi phudinhgioihan on 27-01-2013 - 02:24 trong Giải tích

Bài $23$ :
Cho $\text{f(x)}$ là hàm số xác định và liên tục trên $\left [ 0, 1 \right ]$ và $\left | \text{f(x)} \right | \leqslant 1$ $,$ $\forall x \in \left [ 0, 1 \right ]$.
Chứng minh rằng :
$\int_{0}^{1}\sqrt{1 - \text{f}^{2}\text{(x)}} \text{dx} \leqslant \sqrt{1 - \left ( \int_{0}^{1} \text{f(x)dx} \right )^{2}}$.


Bận mấy bữa để bài này lên men mốc hết rồi :(

Bdt tương đương với

$$\left(\int_0^1 \sqrt{1-f^2(x)}dx \right)^2 +\left(\int_0^1 f(x)dx\right)^2 \le1 $$

$$LHS \underset{C-S}{\le} \int_0^1 dx \int_0^1 (1-f^2 (x) )dx + \int_0^1 dx \int_0^1 f^2(x)dx =\int_0^1dx =1 $$

Vậy có đpcm.



#386180 Topic về Bất đẳng thức trong Tích phân

Đã gửi bởi phudinhgioihan on 13-01-2013 - 02:37 trong Giải tích

Bài 20: (Mới chế :D)

Cho $f : [0;2] \longrightarrow \mathbb{R} $ , $f'$ liên tục trên $[0;2]$ đồng thời $f(2)=0 \;\;, \int_0^2 f(x)dx=\int_0^2 xf(x)dx =k $

Chứng minh : $$\int_0^2 [f'(x)]^2 dx \ge \dfrac{15}{16}k^2$$


Rắc....tự sướng :D

Ta có :

$$\int_0^2f(x)dx=xf(x) |_0^2-\int_0^2xf'(x)dx=-\int_0^2 xf'(x)dx$$

$$\int_0^2 xf(x)dx=\frac{1}{2}x^2f(x) |_0^2-\dfrac{1}{2} \int_0^2 x^2f'(x) dx =-\dfrac{1}{2}\int_0^2 x^2f'(x)dx$$

$$\Rightarrow k=\int_0^2f'(x) (x-x^2)dx$$

$$\Rightarrow k^2 \le \int_0^2 (x-x^2)^2dx \int_0^2 [f'(x)]^2 dx$$

$$\Leftrightarrow k^2 \le \dfrac{16}{15}\int_0^2 [f'(x)]^2dx$$



#392471 Topic về Bất đẳng thức trong Tích phân

Đã gửi bởi phudinhgioihan on 02-02-2013 - 12:45 trong Giải tích

Bài 28: Cho $f:[0,1]\to\mathbb{R}$ là hàm khả vi sao cho $f(0)=f(1)=0$ và $|f'(x)|\leq 1,\forall x\in[0,1]$. Chứng minh rằng
$$|\int_0^1 f(t)dt|<\dfrac{1}{4}.$$

Romania-District-Olympiad-2012


$$ \left| \int_0^1 f(x)dx \right| \le \left| \int_0^{\frac{1}{2}} f(x)dx \right| +\left| \int_{\frac{1}{2}}^1 f(x)dx \right|$$

$$\le \left| \int_0^{\frac{1}{2}} \int_0^{x} f'(t)dt dx \right| + \left|\int_{\frac{1}{2}}^1 \left(-\int_x^1 f'(t)dt \right) dx \right| $$

$$ \le \int_0^{\frac{1}{2}} \int_0^{x} |f'(t)|dt dx +\int_{\frac{1}{2}}^1 \int_x^1 |f'(t)|dtdx $$

$$ \le \int_0^{\frac{1}{2}} \int_0^{x} dtdx +\int_{\frac{1}{2}}^1 \int_x^1 dtdx $$

$$\le \int_0^{\frac{1}{2}} xdx+\int_{\frac{1}{2}}^1 (1-x)dx $$

$$ \le \dfrac{1}{4}$$

Dấu $=$ xảy ra khi $|f'(x)|=1 \;\;, \forall x \in [0;1] $ suy ra $f(x)$ là hàm bậc nhất, do đó không thể có $f(0)=f(1)=0$, vậy trường hợp đẳng thức không thể xảy ra.

Vậy $$ \left| \int_0^1 f(x)dx \right| <\dfrac{1}{4}$$